Let $(a_n), (b_n),$ be bounded, then prove that $c_n$ converges and give its value.












3












$begingroup$


Could I get some feedback on the following proof, I feel becoming a good mathematician is through constant feedback and improvement of your work, I tried to make it short, but well-readable.




We are given that $a_n$ and $b_n$ are bounded sequences, and also:
$$ (n-1)a_n leq n^2 c_n leq (n+1)b_n$$
Prove that $c_n$ converges and give the value of the limit.




First of all, we know that these sequences are bounded, so surely we have that:
$$L geq a_n land b_n leq U$$
For some lower bound $L$ and some upper bound $U$ in $mathbb{R}$.
We now apply this:
$$ (n-1) L leq (n-1)a_n leq n^2 c_n leq (n+1)b_n leq (n+1)U$$
$$ frac{L}{n}-frac{L}{n^2}=frac{(n-1) L}{n^2} leq c_nleq frac{(n+1) U}{n^2} =frac{U}{n}+frac{U}{n^2}$$
Both these sequence converge to zero, so we have that in the limit:
$$0 leq lim_{nrightarrow infty} c_n leq 0$$
By the squeeze theorem we have that $lim_{nrightarrow infty} c_n =0$










share|cite|improve this question









$endgroup$








  • 1




    $begingroup$
    Yes, it is correct.
    $endgroup$
    – José Carlos Santos
    Nov 29 '18 at 21:29


















3












$begingroup$


Could I get some feedback on the following proof, I feel becoming a good mathematician is through constant feedback and improvement of your work, I tried to make it short, but well-readable.




We are given that $a_n$ and $b_n$ are bounded sequences, and also:
$$ (n-1)a_n leq n^2 c_n leq (n+1)b_n$$
Prove that $c_n$ converges and give the value of the limit.




First of all, we know that these sequences are bounded, so surely we have that:
$$L geq a_n land b_n leq U$$
For some lower bound $L$ and some upper bound $U$ in $mathbb{R}$.
We now apply this:
$$ (n-1) L leq (n-1)a_n leq n^2 c_n leq (n+1)b_n leq (n+1)U$$
$$ frac{L}{n}-frac{L}{n^2}=frac{(n-1) L}{n^2} leq c_nleq frac{(n+1) U}{n^2} =frac{U}{n}+frac{U}{n^2}$$
Both these sequence converge to zero, so we have that in the limit:
$$0 leq lim_{nrightarrow infty} c_n leq 0$$
By the squeeze theorem we have that $lim_{nrightarrow infty} c_n =0$










share|cite|improve this question









$endgroup$








  • 1




    $begingroup$
    Yes, it is correct.
    $endgroup$
    – José Carlos Santos
    Nov 29 '18 at 21:29
















3












3








3





$begingroup$


Could I get some feedback on the following proof, I feel becoming a good mathematician is through constant feedback and improvement of your work, I tried to make it short, but well-readable.




We are given that $a_n$ and $b_n$ are bounded sequences, and also:
$$ (n-1)a_n leq n^2 c_n leq (n+1)b_n$$
Prove that $c_n$ converges and give the value of the limit.




First of all, we know that these sequences are bounded, so surely we have that:
$$L geq a_n land b_n leq U$$
For some lower bound $L$ and some upper bound $U$ in $mathbb{R}$.
We now apply this:
$$ (n-1) L leq (n-1)a_n leq n^2 c_n leq (n+1)b_n leq (n+1)U$$
$$ frac{L}{n}-frac{L}{n^2}=frac{(n-1) L}{n^2} leq c_nleq frac{(n+1) U}{n^2} =frac{U}{n}+frac{U}{n^2}$$
Both these sequence converge to zero, so we have that in the limit:
$$0 leq lim_{nrightarrow infty} c_n leq 0$$
By the squeeze theorem we have that $lim_{nrightarrow infty} c_n =0$










share|cite|improve this question









$endgroup$




Could I get some feedback on the following proof, I feel becoming a good mathematician is through constant feedback and improvement of your work, I tried to make it short, but well-readable.




We are given that $a_n$ and $b_n$ are bounded sequences, and also:
$$ (n-1)a_n leq n^2 c_n leq (n+1)b_n$$
Prove that $c_n$ converges and give the value of the limit.




First of all, we know that these sequences are bounded, so surely we have that:
$$L geq a_n land b_n leq U$$
For some lower bound $L$ and some upper bound $U$ in $mathbb{R}$.
We now apply this:
$$ (n-1) L leq (n-1)a_n leq n^2 c_n leq (n+1)b_n leq (n+1)U$$
$$ frac{L}{n}-frac{L}{n^2}=frac{(n-1) L}{n^2} leq c_nleq frac{(n+1) U}{n^2} =frac{U}{n}+frac{U}{n^2}$$
Both these sequence converge to zero, so we have that in the limit:
$$0 leq lim_{nrightarrow infty} c_n leq 0$$
By the squeeze theorem we have that $lim_{nrightarrow infty} c_n =0$







real-analysis sequences-and-series limits proof-verification






share|cite|improve this question













share|cite|improve this question











share|cite|improve this question




share|cite|improve this question










asked Nov 29 '18 at 21:15









Wesley StrikWesley Strik

1,653423




1,653423








  • 1




    $begingroup$
    Yes, it is correct.
    $endgroup$
    – José Carlos Santos
    Nov 29 '18 at 21:29
















  • 1




    $begingroup$
    Yes, it is correct.
    $endgroup$
    – José Carlos Santos
    Nov 29 '18 at 21:29










1




1




$begingroup$
Yes, it is correct.
$endgroup$
– José Carlos Santos
Nov 29 '18 at 21:29






$begingroup$
Yes, it is correct.
$endgroup$
– José Carlos Santos
Nov 29 '18 at 21:29












1 Answer
1






active

oldest

votes


















1












$begingroup$

Absolutely fine proof. You may want to find if the given condition can be relaxed. For example, if $n^2$ is replaced by $n^alpha$ in the above proof, which values would work? $n = n^1$ would not work : try to find a counterexample.






share|cite|improve this answer









$endgroup$













  • $begingroup$
    Nice variation on the problem :)
    $endgroup$
    – Wesley Strik
    Nov 29 '18 at 23:36










  • $begingroup$
    You are welcome!
    $endgroup$
    – астон вілла олоф мэллбэрг
    Nov 30 '18 at 1:20











Your Answer





StackExchange.ifUsing("editor", function () {
return StackExchange.using("mathjaxEditing", function () {
StackExchange.MarkdownEditor.creationCallbacks.add(function (editor, postfix) {
StackExchange.mathjaxEditing.prepareWmdForMathJax(editor, postfix, [["$", "$"], ["\\(","\\)"]]);
});
});
}, "mathjax-editing");

StackExchange.ready(function() {
var channelOptions = {
tags: "".split(" "),
id: "69"
};
initTagRenderer("".split(" "), "".split(" "), channelOptions);

StackExchange.using("externalEditor", function() {
// Have to fire editor after snippets, if snippets enabled
if (StackExchange.settings.snippets.snippetsEnabled) {
StackExchange.using("snippets", function() {
createEditor();
});
}
else {
createEditor();
}
});

function createEditor() {
StackExchange.prepareEditor({
heartbeatType: 'answer',
autoActivateHeartbeat: false,
convertImagesToLinks: true,
noModals: true,
showLowRepImageUploadWarning: true,
reputationToPostImages: 10,
bindNavPrevention: true,
postfix: "",
imageUploader: {
brandingHtml: "Powered by u003ca class="icon-imgur-white" href="https://imgur.com/"u003eu003c/au003e",
contentPolicyHtml: "User contributions licensed under u003ca href="https://creativecommons.org/licenses/by-sa/3.0/"u003ecc by-sa 3.0 with attribution requiredu003c/au003e u003ca href="https://stackoverflow.com/legal/content-policy"u003e(content policy)u003c/au003e",
allowUrls: true
},
noCode: true, onDemand: true,
discardSelector: ".discard-answer"
,immediatelyShowMarkdownHelp:true
});


}
});














draft saved

draft discarded


















StackExchange.ready(
function () {
StackExchange.openid.initPostLogin('.new-post-login', 'https%3a%2f%2fmath.stackexchange.com%2fquestions%2f3019230%2flet-a-n-b-n-be-bounded-then-prove-that-c-n-converges-and-give-its-val%23new-answer', 'question_page');
}
);

Post as a guest















Required, but never shown

























1 Answer
1






active

oldest

votes








1 Answer
1






active

oldest

votes









active

oldest

votes






active

oldest

votes









1












$begingroup$

Absolutely fine proof. You may want to find if the given condition can be relaxed. For example, if $n^2$ is replaced by $n^alpha$ in the above proof, which values would work? $n = n^1$ would not work : try to find a counterexample.






share|cite|improve this answer









$endgroup$













  • $begingroup$
    Nice variation on the problem :)
    $endgroup$
    – Wesley Strik
    Nov 29 '18 at 23:36










  • $begingroup$
    You are welcome!
    $endgroup$
    – астон вілла олоф мэллбэрг
    Nov 30 '18 at 1:20
















1












$begingroup$

Absolutely fine proof. You may want to find if the given condition can be relaxed. For example, if $n^2$ is replaced by $n^alpha$ in the above proof, which values would work? $n = n^1$ would not work : try to find a counterexample.






share|cite|improve this answer









$endgroup$













  • $begingroup$
    Nice variation on the problem :)
    $endgroup$
    – Wesley Strik
    Nov 29 '18 at 23:36










  • $begingroup$
    You are welcome!
    $endgroup$
    – астон вілла олоф мэллбэрг
    Nov 30 '18 at 1:20














1












1








1





$begingroup$

Absolutely fine proof. You may want to find if the given condition can be relaxed. For example, if $n^2$ is replaced by $n^alpha$ in the above proof, which values would work? $n = n^1$ would not work : try to find a counterexample.






share|cite|improve this answer









$endgroup$



Absolutely fine proof. You may want to find if the given condition can be relaxed. For example, if $n^2$ is replaced by $n^alpha$ in the above proof, which values would work? $n = n^1$ would not work : try to find a counterexample.







share|cite|improve this answer












share|cite|improve this answer



share|cite|improve this answer










answered Nov 29 '18 at 23:22









астон вілла олоф мэллбэргастон вілла олоф мэллбэрг

37.5k33376




37.5k33376












  • $begingroup$
    Nice variation on the problem :)
    $endgroup$
    – Wesley Strik
    Nov 29 '18 at 23:36










  • $begingroup$
    You are welcome!
    $endgroup$
    – астон вілла олоф мэллбэрг
    Nov 30 '18 at 1:20


















  • $begingroup$
    Nice variation on the problem :)
    $endgroup$
    – Wesley Strik
    Nov 29 '18 at 23:36










  • $begingroup$
    You are welcome!
    $endgroup$
    – астон вілла олоф мэллбэрг
    Nov 30 '18 at 1:20
















$begingroup$
Nice variation on the problem :)
$endgroup$
– Wesley Strik
Nov 29 '18 at 23:36




$begingroup$
Nice variation on the problem :)
$endgroup$
– Wesley Strik
Nov 29 '18 at 23:36












$begingroup$
You are welcome!
$endgroup$
– астон вілла олоф мэллбэрг
Nov 30 '18 at 1:20




$begingroup$
You are welcome!
$endgroup$
– астон вілла олоф мэллбэрг
Nov 30 '18 at 1:20


















draft saved

draft discarded




















































Thanks for contributing an answer to Mathematics Stack Exchange!


  • Please be sure to answer the question. Provide details and share your research!

But avoid



  • Asking for help, clarification, or responding to other answers.

  • Making statements based on opinion; back them up with references or personal experience.


Use MathJax to format equations. MathJax reference.


To learn more, see our tips on writing great answers.




draft saved


draft discarded














StackExchange.ready(
function () {
StackExchange.openid.initPostLogin('.new-post-login', 'https%3a%2f%2fmath.stackexchange.com%2fquestions%2f3019230%2flet-a-n-b-n-be-bounded-then-prove-that-c-n-converges-and-give-its-val%23new-answer', 'question_page');
}
);

Post as a guest















Required, but never shown





















































Required, but never shown














Required, but never shown












Required, but never shown







Required, but never shown

































Required, but never shown














Required, but never shown












Required, but never shown







Required, but never shown







Popular posts from this blog

Plaza Victoria

In PowerPoint, is there a keyboard shortcut for bulleted / numbered list?

How to put 3 figures in Latex with 2 figures side by side and 1 below these side by side images but in...